GMAT Data Sufficiency Warm-up Practice

Manhattan Elite Prep's origin can be traced to an Ivy-League classroom in 1996. Since the inception, we have grown into a multi-national firm, focusing on helping you achieve the highest GMAT, GRE, TOEFL, SAT, LSAT & MCAT scores with the least amount of time and financial investments. Additionally, we offer assistance with college, business and graduate school admissions consulting, application advisory and essay editing services. We also offer private tutoring for all K-12 academic subjects including math, English, history and more. Our GMAT Prep services include in-person GMAT Prep group classes, GMAT Prep online courses (and online recordings), GMAT Prep private tutoring & 1-on-1 GMAT Prep private courses.

Answer choices for Data Sufficiency questions Q1-10:

(A) Statement (1) ALONE is sufficient, but statement (2) alone is not sufficient.

(B) Statement (2) ALONE is sufficient, but statement (1) alone is not sufficient.

(C) BOTH statements TOGETHER are sufficient, but NEITHER statement ALONE is sufficient.

(D) EACH statement ALONE is sufficient

(E) Statements (1) and (2) TOGETHER are NOT sufficient.

Q1. A, B, C, D are four separate figures. Is CD > BC?

(1) AD= 20

(2) AB = CD

Solution: Knowing that AD= 20 makes statement one as insufficient. We have to think about evaluating the statements above in combination. They are not sufficient in isolation nor are they sufficient in combination.

The correct answer is (E).

Q2. How many more men than women are in the room?

(1) There is a total of 20 women and men in the room.

(2) The number of men in the room equals the square of the number of women in the room.

Solution: In isolation, statement one is insufficient. If we evaluate them in combination, we can plug in numbers and see if that works.

The correct answer is (C).

Q3. If n is an integer, is (100-n)/n an integer?

(1) n > 4

(2) n^2 = 25

Solution: (100-5)/5, but what if n = 6 : (100-6)/6? We have one case where we get an integer and the other where it is not an integer.

What about statement two? If n^2 = 25 then there are two possible values for n, n can equal positive or negative 5. If we plug in those numbers then both possible values of n give you an integer.

The correct answer is (B).

Q4. Last Friday a shop sold ¾ of the sweaters in its inventory. Each sweater sold for $20. What was the total revenue last Friday from the sale of these sweaters?

(1) When the shop opened last Friday there were 160 sweaters in its inventory.

(2) All but 40 sweaters in the shop's inventory were sold last Friday.

Solution: Statement one is sufficient. Statement two is also sufficient.

Therefore the correct answer is (D).

Q5. A jar contains 30 marbles, of which 20 are red and 10 are blue. If 9 of the marbles are removed, how many of the marbles left in the jar are red?

(1) Of the marbles removed, the ratio of the number of red ones to the number of blue ones is 2:1.

(2) Of the first 6 marbles removed, 4 are red.

Solution: Statement one is sufficient. Statement 2 is not sufficient because we don't know about the remaining two marbles' colors.

The correct answer is (A).

Q6. If w + z = 28, what is the value of wz?

(1) w and z are positive integers.

(2) w and z are consecutive odd integers.

Solution: Statement 1 is not sufficient. The only possibly consecutive odd integers are 13 and 15, and if you know this then you can work out the value of w times z.

The correct answer is (B).

Q7. Is ax = 3 - bx?

(1) x(a + b) = 3

(2) a = b = 1.5 and x = 1

Solution: Generally speaking, you don't want to do that much scratch work, but this question does require scratch work. x(a + b) = 3 à ax + bx = 3 à ax = 3-bx, so statement one is sufficient in isolation.

Statement two is also sufficient in isolation, therefore the correct answer is (D).

Q8. What is the value of the integer x?

(1) x is a prime number.

(2) 31 ≤ x ≤ 37

Solution: Statement one is not sufficient in isolation and statement two in isolation is also not sufficient. In combination, 31 and 37 are prime numbers, so statements one and two are insufficient.

The correct answer is (E). You need to know all of the prime numbers up to 37 for the GMAT.